Forums

Looking for impossible chess positions

Sort:
GeertDD

Find the mistake on the board. What makes the position impossible?

Can you think of other setups which maybe are a bit more interesting? Thanks.

GeertDD

Lol, that's too easy, harryz. Here's another one:

CP6033

this one is easy!

CP6033

how about this oneÉ

CP6033

didn`t work was supposed to say black to move

CP6033

Whatever happened to white moves first 9000700.

TheMushroomDealer

#6 is possible if it is White's or Black's move

TheMushroomDealer

For example 1.Nf3 d6 2.Ng1 d5

TheMushroomDealer

1.Nf3 d5 2.Ng1 Be6 3.Nf3 Bc8 4.Ng1

chessdex
CP6033 wrote:

how about this oneÉ

 

black to move in the starting position is possible. Move the knights

BigDoggProblem
caveatcanis
chessdex wrote:
CP6033 wrote:

how about this oneÉ

 

black to move in the starting position is possible. Move the knights

You can reach the starting position only after an even number of moves, and with White to play.

Knights can't lose/gain a move. The one square vacated by a knight isn't enough for the adjacent rook to lose a move either.

chaotic_iak

#17: Wow, this problem is fantastic. Black needs to capture White's dark bishop on its home (b2 and d2 aren't moved). In addition, there must be gxf6. Finally, that bishop at h1 can only be the result of c7-d6-e5-f4-g3-h2-h1=B, performing five captures. These all count for all seven captures made by Black. But then White's light bishop cannot be captured (all capture squares are dark).

chaotic_iak

caveatcanis

@chaotic_jak Whose move is it in #20?

Remellion

#20 - White's sextupled pawns come from the a- through f-files and made 15 captures, all of black's army. Therefore it's black's move, since he can't retract anything. (...Kg7-h8 impossible since white can't retract g5xf6 as that pawn came from e5.)

Now we look to retract a white move; it can't be either knight or the king capturing, as they wouldn't lift the stalemate, and all captures were made by pawns. It seems like retracting any pawn capture gives black all the necessary space to move, so that deals with the immediate problem. It looks constructible; the easiest proof is in the pudding, so:



Remellion

A triplet! Are the following legal? 1) as per the diagram; 2) with the g2-pawn on g3; 3) with the g2-pawn on h3.

computo200

@23 this looks like a nice one.

1) Black bishop at a6 position seems legal. d pawn, has moved from d file so bishop is free to go ( nearly ) anywhere. All pawn-positions in the board are legal. All other bishops are legal, and the O-Os existant look 100% legal as well. It looks like the above thought that the missing rook at h is an impossible situation, as the bishop hasn't moved. However the black horses could have captured the rooks and then moved back to the spot they are. However, for the sake of the person who made this, let's suppose those black knights made only one move.

In this case, 1) is impossible as the rook at h1 can't be gone.

2) the bishop could have moved out, following a castling , then the rook could have left from c file and king could go back to his original spot.

3) same, or rook could have left immedietally.

 

Thats a very nice one! However, if you hadnt missed the knight thing... :P

 

UPDATED :D

Remellion

^ Wrong reasoning and wrong answer. I didn't miss any knight thing, you're thinking too simply. If you really think all 3 are legal, show me the proof games, without your self-imposed knight restriction. You may find some issue with the bishop...

That's why I like constructing positions that look plausible but are trickier. Always the chance to catch someone out.

tmkroll

White's only missing two rooks. With the pawn on g2 the King's Rook was taking on its home square. The other rook was taken on c6 after c4 was pushed. The Black Bishop could only reach a6 if the capture on c6 happened before c4 was pushed, so it's not legal. With the pawn on g2 both rooks could have gotten out after the c4 push, but I don't see what that changes unless I'm missing something. With the pawn on h3 the King's Rook could have gotten out and been captured on c6 before the c4 push giving the Black Bishop time to get into position, but the pawn could not have made it to h3. Black is only missing the a pawn and the dark square Bishop. The Bishop cannot have been captures on the light square h3, and the pawn can't have made it that far either.